Information transfer using entanglement?

In summary, the conversation centered around the topic of using entanglement to transfer information. It was discussed that entanglement cannot be used to send faster-than-light information, as information from both parties is required. However, it was also mentioned that entanglement can be used to encode more information on a single particle than is physically possible without it. The conversation then shifted to a specific case where the outcomes at B were not affected by variations in the setting at A, and the validity of this scenario was debated. Ultimately, it was agreed that this type of substitution may be valid in a local hidden variable model, but it is not clear if it is valid in other models.
  • #1
entropy1
1,230
71
I split this off a separate thread in response to a post of @Nugatory .
The matter was about the (im)possibility of transfering information using entanglement.
This is a basic thread, so I keep it simple: There are two particles/detectors, A and B. The particles are in the singlet state.
Nugatory said:
No. It does prove that measurements on an entangled system do not transmit information at any speed, whether faster than light or not.
My question:
Could we claim that, should we have chosen a different detector angle for A, measurement B would have had the same outcome?
If we can't claim that, can we even rule out causation, and in that context, information transfer?
Of course any change in A doesn't affect the randomness of the outcomes of B. So, if the random pattern in B is changed in a different random pattern by changing something in A, there is no information transfer. There is an effect however (in this supposition).

My train of thought went like this: suppose a single pair in singlet state does not facilitate information transfer. However, a change in angle of A could lead to a different result in B. Then would it be conceivable that an ensemble of pairs (in time of space) would lead to a whole pattern of deviations in B as an effect of changes in the angle of detector A? I'm thinking for instance that the pattern A shows correlates with the pattern B shows (which is manifest in singlet ensembles). There is, as it were, information about a relative relation.

It may be a long shot and I'm won't be too surprised if I overlook something. :biggrin:
 
Last edited:
Physics news on Phys.org
  • #2
Without receiving information from A, there is no way of telling from the ensemble of B, whether B is entangled with A or not. Information from both parties is required, which is why entanglement cannot be used to send FTL information.

Entanglement can still be used to send information, such as the case with quantum super-dense coding.
Using entanglement, one can encode twice as much information on a particle than is physically possible without entanglement.
This is because manipulating A alone can change the entangled state A and B share to be any possible Bell state.
Since there are more distinguishable Bell states of a pair of particles than there are eigenstates of a single particle, one can send more information using entanglement than one can without it.
 
  • #3
jfizzix said:
Without receiving information from A, there is no way of telling from the ensemble of B, whether B is entangled with A or not. Information from both parties is required, which is why entanglement cannot be used to send FTL information.
I follow that. On one side (A or B) the measurements seem random. What I am suggesting is that the actual manifestation of such a random pattern may be dependent of the angle of the detector, for if that wouldn't be the case, correlations wouldn't be able to vary between A and B dependent of the angle. A and B don't know (can't verify) that they correlate, but they do! When A and B meet each other they could say: "Hey, we correlate!" as they were doing all along. This information is relative, like it would for instance be relative to find you both have a paper with letters that combined make a message (i.e. the correlation). The information in question is as it were distributed over A and B. What I mean is that changing one of the detector angles changes the correlation, and so something has to have changed in one or both random patterns.
 
Last edited:
  • #4
How much we can delay light?

Let polarization be entangled(E) and be one part thru angled slit(S) and other go undisturbed(U).
If we increase time E-S can we "decrease" distance S-U?
 
  • #5
The following is a case that outcomes at B are not affected as setting at A varies:
Trial 1.
A, α = 30o
B, β = 1700
In this trial θ = 1400 With the sin2 formula calculate P++ and P-- = .883
Trial 2.
A,α = 500
B,β = 1700
With data outcomes from trial 1 detector A (30o) and data outcomes at trial 2 detector B 1700 )
determine by inspection if P++ P-- also = .883
and therefore B outcomes at 1700 were not affected as A setting varied from 300 to 500

And furthermore if the above is valid and two more trials show that A outcomes were not affected as B
setting varied. Then this could be a case for E (a,b) = ∫ dλ C (a.a' b,b' λ)
 
Last edited:
  • #6
morrobay said:
With data outcomes from trial 1 detector A (30o) and data outcomes at trial 2 detector B 1700 )
determine by inspection if P++ P-- also = .883
and therefore B outcomes at 1700 were not affected as A setting varied from 300 to 500
I follow that, but I am not sure I agree. If you would substitute Trial 2 (α=50°) with Trial 1 (α=30°) then Trial 2 would get a different correlation. A specific correlation occurs between two quasi random binary patterns only at certain values at a certain alignment (and I have code to back this up). If you have a pair of (quasi random) strings of values and you replace one of the pair by a different string, you most probably get a non-correlating pair (ie random p=.5).
 
Last edited:
  • #7
Ok, see post #21 by @Nugatory in my thread: Local hidden variable model that equals qm predictions. The second part of reply for my sidetrack question. Unless there was a misunderstanding @Nugatory agreed with these types of substitutions being valid.
 
  • #8
morrobay said:
In other words is it valid for this particular model to combine an outcome at setting at A , 800 from one pair. And then from another pair an outcome at B , at 3330 . To say this way: in this model could the results from two different pairs, A at 800 and B at 3330 be equal to outcomes for one pair measured at A, 800 and B 3330 ? If this is invalid then all 3602 measurements could be made.
I am not sure what you are saying here...

If you are suggesting taking measurements at one side of an entangled pair (say, A), and later on combine that with an independent measurement on the other side (B), I think you get two uncorrelated measurements.
 
Last edited:
  • #9
entropy1 said:
I am not sure what you are saying here...
Just the same scenario as in post 5 above : trials 1 , 2
 
  • #10
Since B1 and B2 have a different dataset, replacing A2 with A1 in set 2 will affect the correlation. The set 1 correlation is characteristic for A1 together with B1.

That's how I see it. :smile:
 
  • #11
morrobay said:
The following is a case that outcomes at B are not affected as setting at A varies:
Trial 1.
A, α = 30o
B, β = 1700
In this trial θ = 1400 With the sin2 formula calculate P++ and P-- = .883
Trial 2.
A,α = 500
B,β = 1700
With data outcomes from trial 1 detector A (30o) and data outcomes at trial 2 detector B 1700 )
determine by inspection if P++ P-- also = .883
and therefore B outcomes at 1700 were not affected as A setting varied from 300 to 500

And furthermore if the above is valid and two more trials show that A outcomes were not affected as B
setting varied. Then this could be a case for E (a,b) = ∫ dλ C (a.a' b,b' λ)

I am essentially in agreement with entropy1's comments on the above. Clearly the observed statistics change (a suitable set of trials) as theta changes for entangled pairs. This is not in question, is it? And if it's not, I don't follow the point either.

As to whether B outcomes are affected by varying the angle A is measured at: this is interpretation dependent. No one understands the precise mechanics.
 
  • #12
DrChinese said:
As to whether B outcomes are affected by varying the angle A is measured at: this is interpretation dependent. No one understands the precise mechanics.
Ok. My question primarily targeted the possibility that for a single pair, if we we would have chosen a different measurement angle for A, the outcome could have been different for B. I think this is not exactly CFD because we do measure B. There is however no verifying both of A's angles in one trail. But, as you point out, the statistics (of the combination of A and B) are dependent of the theta angle, and I observe that the theta angle can only change if one or both angles of A and B change, so when we change angle alpha there should be a change in either measurement ensemble A and/or measurement ensemble B. You could argue that if you change angle alpha, that only measurements of A change. However, that change should be in accordance with the correlation, and thus with theta, and thus A's measurements should depend on B's measurements, so then we have non-locality. Since the situation is symmetric, you could also associate the change in theta to B's measurements. So indeed we can't pinpoint the underlying mechanics. But having said that, in my view it is possible that angle alpha has 'an' effect on B's measurement outcomes.
 
  • #13
entropy1 said:
... Since the situation is symmetric, you could also associate the change in theta to B's measurements. So indeed we can't pinpoint the underlying mechanics. But having said that, in my view it is possible that angle alpha has 'an' effect on B's measurement outcomes.

Sure, probably so, in fact how could it not if the stats change? But again, there are interpretations in which there is no physical collapse. And in those, essentially, there may be no mutual (symmetric) collapse occurring.
 
  • #14
entropy1 said:
Ok. My question primarily targeted the possibility that for a single pair, if we we would have chosen a different measurement angle for A, the outcome could have been different for B. I think this is not exactly CFD because we do measure B. There is however no verifying both of A's angles in one trail. But, as you point out, the statistics (of the combination of A and B) are dependent of the theta angle, and I observe that the theta angle can only change if one or both angles of A and B change, so when we change angle alpha there should be a change in either measurement ensemble A and/or measurement ensemble B. You could argue that if you change angle alpha, that only measurements of A change. However, that change should be in accordance with the correlation, and thus with theta, and thus A's measurements should depend on B's measurements, so then we have non-locality. Since the situation is symmetric, you could also associate the change in theta to B's measurements. So indeed we can't pinpoint the underlying mechanics. But having said that, in my view it is possible that angle alpha has 'an' effect on B's measurement outcomes.
Hopefully making things more definite will add clarity.

1) From B's perspective he will see ±1 with probability ½ for each no matter what A is doing. Same for A. We know if they both measure at 0º they get the same value.

2) If A measures at 45º and gets -1, while B measures at 0º and gets 1 what would they get if A measured at 0º instead? Would they both get 1? -1? If you insist that A must get 1 you are assuming she already had a value at 0º - CFD. However that experiment was not made, so we don't know.

How do you suggest in this context information might be exchanged?
 
  • Like
Likes entropy1
  • #15
entropy1 said:
You could argue that if you change angle alpha, that only measurements of A change. However, that change should be in accordance with the correlation, and thus with theta,

Following is data set/model suggesting that for two spin 1/2 particles in superposition with spin state l Ψ > = 1/√2 [ l +,-> -l -, + > ]
Setting/outcome changes at A do not have effect on B outcomes.* While outcomes at A are independent of setting at B , outcomes at A are not independent of outcomes at B ( the correlations ) Measurement order can be A , B or B, A
A 00 ..... B 1200_______________ * (sin θ/2)2 P--, P++ = .75
- ....... -
+........-
+........+
-.......-
-.......+
+........+
+........+
+........+


A 600.....B 1200_______________* (sinθ/2)2 = P--. P++ =.25 * (1/2 omitted for simplicity)
+......-
+......-
-......+
-......-
-......+
-......+
+......+
-......+

Voila !

*
the only way information could be transferred is if outcome at A is dependent on setting at B and visa versa
 
Last edited:
  • Like
Likes entropy1
  • #16
@morrobay Your example is a perfect illustration of your quote of my text. So indeed that is a possibility. However, as I pointed out, A's outcomes must conform to theta and to B's outcomes, and hence can't be local. And my point was that since they can't be local, changes in A might just as well bring about changes in B, that being non-local too.
 
  • #17
Zafa Pi said:
How do you suggest in this context information might be exchanged?
To clear this up: The title of my topic may be misleading - I am not advocating information transfer via entanglement, to the contrary: I am only suggesting the information in play, being the correlation between A's and B's outcomes, is dispersed (distributed), and as such dependent of A's and B's outcomes. If theta has a value of θ, the A's an B's outcomes have to 'conspire' to produce the corresponding correlation for θ. So this 'conspiration' I see as a form of 'information exchange', or perhaps better: 'information self-consistency'.
 
  • #18
That outcomes at A and B are dependent or conform only relates to the distant correlations. This is not the same as one electron being affected by the measurement of the other
 
  • Like
Likes entropy1
  • #19
morrobay said:
That outcomes at A and B are dependent or conform only relates to the distant correlations. This is not the same as one electron being affected by the measurement of the other
What if I propose that in your table, given θ (p=.75), on one line we flip one sign on whichever side. To keep the correlation intact, we have to flip the sign on the other side too, right?
 
  • #20
The tables are just abreviations//averages for long stream ensembles . Can QM even address single interactions in these non classical correlations? You have to ask some of the talent here at A level going any farther with this.
 
  • Like
Likes entropy1
  • #21
morrobay said:
The tables are just abreviations//averages for long stream ensembles .
Right! But I can see that as a greater-ensemble variation of the same thing (what I have called: 'conspiracy'): the conspiracy being dispersing information about theta between A and B.
 
  • #22
Zafa Pi said:
2) If A measures at 45º and gets -1, while B measures at 0º and gets 1 what would they get if A measured at 0º instead? Would they both get 1? -1? If you insist that A must get 1 you are assuming she already had a value at 0º - CFD. However that experiment was not made, so we don't know.
I see your point: in your case, if B would be -1, A cannot other than be -1 too, so the outcome of A would have been unchanged. And how can something that did not change have an effect, right? But I'm not speaking of outcomes of A, but of settings of A. Given a different setting at A, would that allow for a different outcome of B?
 
Last edited:
  • #23
I would suggest mathematical outcomes are reliant on symmetries. However, measurement breaks these symmetries.
 
  • #24
If partial signal is measured can any of the rest remain undisturbed?

Is there a difference if part through a slit is later measured or not?
 
  • #25
DrChinese said:
But again, there are interpretations in which there is no physical collapse. And in those, essentially, there may be no mutual (symmetric) collapse occurring.
I am not necessarily speaking of collapse - I am speaking of "measurements". So the issue may 'come down' to "the measurement problem", which essentially is the same thing in a different guise I think...
 
  • #26
entropy1 said:
I see your point: in your case, if B would be -1, A cannot other than be -1 too, so the outcome of A would have been unchanged. And how can something that did not change have an effect, right? But I'm not speaking of outcomes of A, but of settings of A. Given a different setting at A, would that allow for a different outcome of B?
You can not try different settings at A in a single given experiment. Whatever the setting at A, B will get 1 or -1 with probability 1/2.
I suspect that the above sentence does not satisfy your concern, but I still don't know why.
 
  • #27
Zafa Pi said:
You can not try different settings at A in a single given experiment. Whatever the setting at A, B will get 1 or -1 with probability 1/2.
I suspect that the above sentence does not satisfy your concern, but I still don't know why.
There is a caveat to go with that, I suspect: A's and B's outcomes are not independent. They produce a correlation (generally). However, both A and B produce a practically random pattern that we can't influence, so we can't manipulate their outcomes. What we can manipulate however, is the settings (orientations of apparata). My proposal was that, so that de correlation stays satisfied, we have different random patterns at different settings. So my question occurred: do the random patterns change, and which of them? At different settings, either A has to adapt do B, B to A, or something else I don't get. In any case the local outcomes are surely one thing: random. So we can't directly observe a possible change in the patterns. My point was that something in the patterns has to adapt to the correlation, and why not B, when changing A? So I'm not saying that this is the case, but that the information we can extrude of the experiment does not prohibit it!

I am not defending information transfer using entanglement, but only because the experiment stays silent on it. So that leaves the possibility of 'influence'.
 
Last edited:
  • #28
entropy1 said:
Given a different setting at A, would that allow for a different outcome of B?

The tables from this paper address your question: Table 1 shows perfect correlations for twin state photons with parallel polarizers at detectors A and B both at 00
Table 2 shows data from detector A at again 00 and detector B now at 300 and correlations in accord with cos ( β-α)2 = .75
Note that with setting and outcome changes at B , data is un affected at A . Also note this is similar to my post # 15 above,
http://image.ibb.co/n1B4cQ/img009.jpg

Click on above then click again on image to enlarge
The tables are taken from this paper pages 7 & 8
https://arxiv.org/pdf/0902.3827.pdf
Also at: Am. J. Phys, Vol. 78, No 1. January 2010
 
Last edited:
  • #29
Thanks, @morrobay, I will read it later.

I thought of the following: Suppose you have filter A at 90°, and filter B at 0°. Suppose you measure a singlet pair '-1' at A and '1' at B.

Now, if we would have measured both A and B at 0°, either both would have to measure -1, or both would have to measure 1.

This means that either A or B would have to change sign. Now why would there be a preference for A or for B? So, B could have changed, however, it could have been due to random chance!
 
  • #30
If we assume outcomes of A and B are resembling randomness, then, if we pick the angle between A and B so that there is no correlation, one could observe:
  1. All B's are randomly distributed over A='0',
  2. All B's are randomly distributed over A='1'.
However, if we pick the angle so that there is 'more correlation', then:
  1. There are relatively more B='0' than B='1' given A='0',
  2. There are relatively more B='1' than B='0' given A='1'.
Similarly for anti-correlation and also for swapped A and B.

If I measure A='1', then in the second scheme there is more likelyhood for B to measure '1' than to measure '0'. So in the subset of A='1', B is not purely random anymore. However, we can't influence the outcome of A, so we see randomness at B. But could we say there is a pressure on B to consort with outcomes of A, in which we only have a say by increasing or decreasing the pressure by changing the angle?
 
Last edited:
  • #31
@morrobay: thanks for your example in post #15 and the example in the paper. I am sorry, with all due respect, but I am not sure what you are saying here. Are you saying that the outcomes are interdependent, and that they are not dependent on the settings?

Then I would agree, with the caveat that the outcomes may depend on the settings. :smile:

I think that there is - in principle - not enough information to establish how the outcomes are created. The situation is symmetrical, so it could go either way. B depends on A and vice versa. However, that is my point: because there is not enough information in principle, that leaves room for the possibility that B (outcomes) depends on A (outcomes/settings). However, because the opposite is also possible, it seems difficult if not impossible to substantiate a claim of causality or even influence.

That said, I think the outcomes depend on the settings, because the correlation depends on the settings. So if one were to rule out 'an effect' (non-locality) one would have to stick with the outcomes depending on the local settings (like you seem to do?). One step further is to suggest that the outcomes might interdepend non-locally (of 'the other' settings), like I do. :smile:
 
Last edited:

1. What is information transfer using entanglement?

Information transfer using entanglement is a method of transmitting information between two or more particles that are entangled, meaning they are in a correlated state where the properties of one particle are dependent on the properties of the other. This allows for instantaneous communication between the particles, regardless of the distance between them.

2. How does entanglement work for information transfer?

Entanglement works by creating a pair of particles with correlated properties, such as spin or polarization. These particles are then separated, but remain connected through a phenomenon known as quantum entanglement. When one particle's state is measured, the other particle's state is instantly determined, allowing for information to be transferred between the two particles.

3. What are the potential applications of information transfer using entanglement?

Information transfer using entanglement has potential applications in quantum communication, cryptography, and computing. It could also be used for secure communication between distant locations, as any attempt to intercept the information would disrupt the entanglement and be immediately detected.

4. What are the limitations of information transfer using entanglement?

One limitation of information transfer using entanglement is that it is currently only possible over short distances, as entanglement is easily disrupted by environmental factors. Additionally, the information being transferred must be encoded onto the particles before they are separated, which can be a complex and challenging process.

5. How is information transfer using entanglement different from traditional communication methods?

Information transfer using entanglement is fundamentally different from traditional communication methods, which rely on the transmission of physical signals. In entanglement-based communication, information is transmitted instantaneously through the entangled particles, without the need for physical signals. This makes it potentially faster and more secure than traditional methods.

Similar threads

  • Quantum Physics
Replies
27
Views
799
Replies
1
Views
817
  • Quantum Physics
Replies
2
Views
270
Replies
41
Views
2K
Replies
33
Views
2K
  • Quantum Physics
Replies
4
Views
981
  • Quantum Physics
Replies
13
Views
1K
  • Quantum Physics
Replies
8
Views
695
Replies
19
Views
2K
Back
Top